Grupo de Lorentz y clasificación de campos por su transformación bajo transformaciones de Lorentz

Tengamos un grupo de Lorentz con generadores de 3 rotaciones, R ^ i , y Lorentz aumenta, L ^ i . Introduciendo operadores

j ^ i = 1 2 ( R ^ i + i L ^ i ) , k ^ i = 1 2 ( R ^ i i L ^ i )

hacemos que el álgebra del grupo de Lorentz sea lo mismo que el grupo SU(2) (o SO(3)). Entonces, cada representación irreducible del grupo de Lorentz se puede construir como

S ^ ( j 1 , j 2 ) = S ^ j 1 × S ^ j 2 ,
dónde j 1 , j 2 son los valores propios máximos de j ^ i , k ^ i ,

y tiene dimensión ( 2 j 1 + 1 ) × ( 2 j 2 + 1 ) . El tipo de objeto, que se transforma a través de impulsos y 3 rotaciones, depende de ( j 1 , j 2 ) :

Ψ α β = S α m j 1 S β v j 2 Ψ m v .
Para ( 0 , 0 ) tenemos escalar, por ( 1 2 , 0 ) , ( 0 , 1 2 ) tenemos espinor (a la izquierda y a la derecha), etc. El valor j 1 + j 2 corresponde al valor máximo de j ^ i + k ^ i = R ^ i , por lo que es un valor propio de representante irreducible del operador de 3 rotaciones y corresponde al número de espín.

Pero la representación irreductible del grupo de Lorentz no es unitaria.

Entonces, la pregunta: ¿cómo podemos clasificar los objetos a través de transformaciones usando repeticiones no unitarias?

Si la representación del grupo de Lorentz en el espacio de campos es unitaria o no, no tiene ningún significado físico. Se requiere que la representación del grupo de Lorentz en el espacio de estados sea unitaria. El espacio de estados es un espacio de Fock generado por los modos de campos de Fourier y, aunque los campos mismos están bajo una representación de dimensión finita (por lo tanto, no unitaria) del grupo de Lorentz, el espacio de Fock generado por sus modos de Fourier da una representación unitaria del grupo de Lorentz.
@usuario10001 . ¿Cómo exactamente el espacio de Fock da una representación unitaria del grupo de Lorentz?
@PhysiXxx Esa es una buena pregunta, pero debe hacerla en su propia publicación, en lugar de enterrarla en los comentarios aquí.

Respuestas (1)

Tenga en cuenta que las partículas corresponden a representaciones unitarias irreductibles del grupo de Poincaré (alias grupo de Lorentz no homogéneo), no solo al grupo de Lorentz.

En estas representaciones de Poincaré, los estados están representados por | pag , λ . pag es el impulso.

Consideremos representaciones masivas positivas ( pag 2 = metro 2 , pag o > 0 ) Dejar π = ( metro , 0 ) . Vemos que tenemos la libertad de elegir la polarización, que corresponde a una S 0 ( 3 ) simetría. Mirando las representaciones unitarias de S O ( 3 ) es lo mismo que mirar representaciones de S tu ( 2 )

Aquí, λ es una base estatal para un pequeño grupo S tu ( 2 ) representación s .

Para una traducción, tenemos:

tu ( a ) | pag , λ = mi i PAG . a | pag , λ

para un miembro R del pequeño grupo S tu ( 2 ) , tenemos :

tu ( R ) | π , λ = λ D λ λ ( s ) ( R ) | π , λ

Para cualquier S L ( 2 , C ) matriz A , y para cualquier pag , es posible escribir una expresión :

tu ( A ) | pag , λ = λ D λ λ ( s ) ( W ( pag , A ) ) | Λ a pag , λ
dónde W ( pag , A ) es un S tu ( 2 ) pequeño elemento de grupo (ver fórmula 18 en la referencia citada a continuación para más detalles)

Con todo esto, se obtiene una representación unitaria del grupo de Poincaré.

El "espacio de Fock" es la versión cuántica de estas representaciones, es decir, permite estados de varias partículas.

Consulte las páginas de referencia 4 y 5

[EDITAR] "¿Para los campos no es importante tener una norma definida positiva invariante de Lorentz?"

No. Tomemos por ejemplo las ecuaciones de Dirac para el campo bi-espinor. la representacion es ( 1 / 2 , 0 ) + ( 0 , 1 / 2 ) . Esta no es una representación unitaria. Hay un espinor izquierdo y uno derecho. La transformación podría escribirse:

ψ L , R =→ mi 1 / 2 ( i σ . θ σ . ϕ ) ψ L , R ,

Los parametros θ corresponden a rotaciones, los parámetros ϕ corresponden a impulsos.

Debido a que la parte de impulso no es unitaria, vemos claramente que la representación no es unitaria.

Entonces, esto significa que la expresión bilineal bispinor ψ ψ = ψ L ψ L + ψ R ψ R no se conserva en una transformación de Lorentz [de hecho, por separado, las expresiones bilineales de espinor ψ L ψ L o ψ R ψ R tampoco se conservan]. Recuerde aquí que el ψ , ψ L , ψ R son campos, no "función de onda".

Es esto un problema ? No.

Qué es ψ ( X ) ψ ( X ) ? Es solo (multiplicado por mi ) la densidad de carga de los campos, es decir j 0 ( X )

Entonces, por supuesto, j 0 ( X ) no es un invariante para una transformación de Lorentz, porque es el componente de tiempo de un vector de Lorentz.

El verdadero invariante de Lorentz está aquí: ψ ¯ ( X ) ψ ( X ) = ψ ( X ) γ 0 ψ ( X )

"...Tenga en cuenta que las partículas corresponden a representaciones unitarias irreductibles del grupo de Poincaré (alias grupo de Lorentz no homogéneo), no solo del grupo de Lorentz...", - pero estamos discutiendo sobre los campos, no sobre las funciones de onda. Para los campos no es importante tener una norma definida positiva invariante de Lorentz.
@PhysiXxx: he editado la respuesta